Среднее значение коммутатора в квантовой механике

Предполагать А и Б операторы, А эрмитов, Б антиэрмитовыми, а их коммутатором является тождество, т.е.

[ А , Б ] "=" я .
Обозначая собственные векторы А как | а , так что А | а "=" а | а , у нас есть
(1) а | [ А , Б ] | а "=" а | а
и
(2) а | ( А Б Б А ) | а "=" а а | Б | а а а | Б | а "=" 0 .

Подводя итог, если | а нормализуется, а | а "=" 1 , мы получаем 0 "=" 1 . Где ошибка?

Что вы просите? Не ясно.
@AlbertoNavarro У ОП есть работа, которая, по сути, показывает, что 0 "=" 1 и хочет, чтобы другие разрешили несоответствие
| а > не нормализуется. Связанный: физика.stackexchange.com /q/10230/2451
@Qмеханик. Не совсем.
@DanielC, но вы показываете, что это невозможно нормализовать, не так ли?

Ответы (2)

Это очень красивая задача теории операторов в сепарабельных гильбертовых пространствах. Хитрость заключается в том, что ваш | а не находится в области коммутатора, поэтому ваше уравнение 1) бессмысленно. Точнее, имеем следующую лемму:

Лемма . Пусть C коммутатор C(A, B; D(C)), в том смысле, что:

ф е Д ( С ) Д ( А Б ) Д ( Б А ) , ( А Б Б А ) ф "=" С ф
Предположим далее, что A самосопряжена с непустым точечным спектром. Необходимое условие для того, чтобы собственные векторы A принадлежали D(C), состоит в том, что C отображает каждый из собственных векторов A в его ортогональное дополнение.

Доказательство . Пусть Aϕ = aϕ и ϕ ∈ D(C). Поскольку собственное значение a действительно и поскольку | ф | Б ф | < , имеем равенство ф | С ф "=" ф | ( А Б Б А ) ф "=" 0 . То есть ф | С ф "=" 0 , или ϕ ортогонален Cϕ.

Предположим теперь, что С "=" 1 ЧАС . Тогда, чтобы иметь собственный вектор ф в области коммутатора из предыдущей леммы следует, что ф "=" 0 .

Красивый ответ на очень важный и популярный вопрос среди физиков-теоретиков! Ключевым моментом является то, что «|a⟩ не находится в области определения коммутатора». Спасибо!

@Qmechanic ответил на ваш вопрос в комментариях, но, очевидно, сообщение не усвоилось автоматически. Позвольте мне попытаться проиллюстрировать его обычной демонстрацией, с которой вы, вероятно, сталкивались, изучая использование нотации скобок Дирака.

Короткий ответ заключается в том, что прослеживаемость тождества в правой части вашего коммутационного уравнения для бесконечномерного гильбертова пространства приводит к а | а 1, потому что на самом деле оно единственное. Итак, ваше уравнение 1) в порядке, так как правая сторона равна бесконечности. Но ваше уравнение 2) ошибочно, так как соответствующее выражение включает в себя 0, умножающий более сильную бесконечность, что, опять же, равно бесконечности, как и в 1).

Я проиллюстрирую это с помощью А "=" Икс ^ и Б "=" п ^ / , как в стандартных курсах QM. Поглотить в п ^ чтобы сделать формализм более знакомым.

Начиная со стандартного операторного уравнения [ Икс ^ , п ^ ] "=" я 1 1 , сначала возьмите его недиагональные матричные элементы, прежде чем строить до вашего 2),

Икс | Икс ^ п ^ п ^ Икс ^ | у "=" ( Икс у ) Икс | п ^ | у "=" ( Икс у ) г п   Икс | п п | п ^ | у "=" ( Икс у ) г п   Икс | п п п | у "=" ( Икс у ) 2 π г п   п   е я ( Икс у ) п "=" я ( Икс у ) Икс дельта ( Икс у ) "=" я дельта ( Икс у ) .
Как всегда, Икс | п "=" опыт ( я Икс п )   / 2 π . Проверьте последнее равенство, работая с корректной тестовой функцией. Это тривиально отражает однородность степени -1, дельта ( λ Икс ) "=" дельта ( Икс ) / λ , поэтому продифференцируйте это по λ и установите λ=1 ​​.

То есть выражение расходится для Икс у , как и 1). Важным моментом является то, что по мере уменьшения префактора (xy) матричный элемент, умножающий его, расходится и быстрее .

Поскольку все матричные элементы коммутатора доступны, как указано выше, вы можете восстановить из них исходные операторные уравнения, вставив разрешения тождества с обеих сторон.

Профессор Захос, что а | а 1 не является последовательностью, потому что можно очень хорошо найти два оператора, удовлетворяющих комм. отношение в гильбертовом пространстве, одно из которых является самосопряженным с нормализуемыми собственными векторами. См. раздел 12.2 книги Брайана Холла «Квантовая теория для математиков». Затем остальная часть вашей аргументации становится ошибочной, как только вы вводите дельта-распределение Дирака. Отсюда следует, что вы изначально работали в пространстве распределения положений собственных векторов, в которые вы пытались вставить коммутатор, который вообще не определен в этом пространстве.
? Итак, вы отвергаете то, что работает каждый курс, следующий за книгой Дирака и его обозначениями? Физики никогда не были загнаны в угол ненормируемыми состояниями, плоскими волнами или операторами неследового класса. Рыбаки на пристани почти не обращают внимания на математические доказательства отсутствия рыбы. Благодаря распределению Дирака физики опередили математиков на десятилетия (...на какое-то время).
Извините, но мое уважение к математически обоснованной физике требует отбрасывания источников, не согласующихся с математикой. Да, КМ — это математическая теория, и она существует уже 91 год, с первым участием фон Неймана по предложению Дэвида Гильберта. Ничто не может изменить это. Можно игнорировать математическую строгость в физике, я предпочитаю не делать этого.
Я тоже уважаю математическую обоснованность: природу не подделаешь. Но основные свойства x и p — это то, с чем работает большинство физиков, а не придуманные странные примеры; если только кто-то не найдет ложку дегтя в конкретном релевантном контексте. Я так понимаю, что ОП попросил совета по стандартному парадоксу QM, а не предупреждения «не ходить туда».
@DanielC Вы говорите, что что-либо с использованием дельта-функции Дирака недействительно? Я просто хочу понять, откуда ты.
@AaronStevens действует только в том случае, если используются тестовые функции. В противном случае это просто махание руками, как здесь: physics.stackexchange.com/questions/435513/… или в ответе выше.
@DanielC Cosmas признает это, но все еще обсуждает, как сделать соответствующие выводы. Вам не нужна идеальная математическая строгость, чтобы понять аргумент. Точно так же, как в математике необходимы строгие доказательства, чтобы убедиться, что все верно, но мы все еще можем учиться, просто понимая идею доказательства. Другими словами, математическая строгость очень важна, но я не думаю, что она всегда требуется, когда мы говорим о концепциях или изучаем их, особенно если строгость мешает лучшему обучению.
Вряд ли в приведенном выше есть что-то, чего нет в книге Дирака, включая однородность дельта-функции. Вейль, который ввел фальшивый парадокс, также фактически разрешил его с помощью круглосуточного QM.